LSAT and Law School Admissions Forum

Get expert LSAT preparation and law school admissions advice from PowerScore Test Preparation.

 Administrator
PowerScore Staff
  • PowerScore Staff
  • Posts: 8916
  • Joined: Feb 02, 2011
|
#41262
Complete Question Explanation
(The complete setup for this game can be found here: lsat/viewtopic.php?t=6641)

The correct answer choice is (B)

If the fifth solo is a T, then Template #2 must be in effect:
PT66_J12_Game_#4_#22_diagram 1.png
Because exactly two pianists are determined in this template, the correct response is answer choice (B).
You do not have the required permissions to view the files attached to this post.
 lsatstudier
  • Posts: 49
  • Joined: Oct 24, 2016
|
#30686
Hi,

I'm not sure if I understand what this question is asking. Are they asking for which pianists can be determined, or which solos can be determined? In looking at this, I thought that the four remaining things could be figured out, but clearly that was wrong. How can you determine the pianists when most of the rules focus on modern versus traditional solos?

Thank you very much in advance for your help!
 Adam Tyson
PowerScore Staff
  • PowerScore Staff
  • Posts: 5153
  • Joined: Apr 14, 2011
|
#30717
They are definitely asking about the pianists, Studier, and we can figure out two of the five. Here's how:

We know the 3rd is a T. The question tells us the 5th is also a T. We also know that there are exactly two T's that are consecutive - not 3, not 4, but 2. That tells us the 4th solo must be an M (because if it was a T then there would be three in a row, not two) and the 2nd must be a T (because otherwise there will not be two Ts in a row). The first one is always an M - that's a global inference based on the last rule. So for this question the order of the type of solos is MTTMT.

So how do we determine who plays which one? Two rules will do that for us. The third rule tells us that the 4th solo is either W playing a T or else Z playing an M. Since we know the 4th solo is an M, it has to be played by Z. That's one.

The last rule tells us that there are no Ts at all until W plays at least one M. Since the second solo is a T, it follows that the first solo must be played by W to comply with that rule. That's two.

Who plays the rest? No idea. 2 and 5 cannot be the same, but we don't know who does which. 3 is a total unknown. So the answer is two - we know the pianists for exactly two of the five solos.

Let us know if that clears things up!

Get the most out of your LSAT Prep Plus subscription.

Analyze and track your performance with our Testing and Analytics Package.